Difference between revisions of "2005 PMWC Problems/Problem T10"

(Solutions)
(Solutions)
 
(6 intermediate revisions by 4 users not shown)
Line 3: Line 3:
  
 
== Solutions ==
 
== Solutions ==
'''First Solution:'''
 
 
 
We list all 2 digit primes:
 
We list all 2 digit primes:
  
Line 26: Line 24:
  
  
Picking a 9 would get us:
+
Note that <math>0,2,4,5,6,8</math> can only appear as the first digit. We can construct the solution by appending digits to the right of the current number with the following maps from the current units digit. <math>1 \to 1,3,7,9; 3 \to 1,7; 7 \to 1,3,9; 9 \to 7</math>. These alone give a maximum of <math>11</math> digits (counting the first digit of the first prime). Note that <math>9</math> may only map one time, but is mapped to from both <math>1</math> and <math>7</math>, so <math>9</math> must also be used as the last digit. To obtain <math>12</math> digits, we must let the first digit must be of the aforementioned alternatives.
 
 
97371311
 
 
 
So we pick an 8.
 
 
 
837319737131
 
 
 
That's the greatest.
 
 
 
 
 
 
 
'''Second Solution:'''
 
 
 
We start with 97, which is the largest 2 digit prime.
 
 
 
97
 
 
 
Then we add 9 to get 79, the largest 2 digit prime with tens digit 7.
 
 
 
979
 
 
 
Add 3 to get 93, the largest prime less than 97
 
 
 
9793
 
 
 
Now the largest two digit prime with tens digit 3 is 37. So we add a 7
 
 
 
97937
 
 
 
Now we add another 3 as 79 already exists.
 
 
 
979373
 
  
And proceeding like this will get us
+
We also note that we need to use both the strings <math>131</math> and <math>737</math>, since the <math>3</math>s may only appear twice. Also, the strings <math>17</math> and <math>97</math> must appear, barring <math>7</math> from being the second digit. These together imply that <math>3,7,9</math> cannot be the first digit, so the second digit must be <math>1</math>. We [[Greedy algorithm|greedily]] use <math>61</math> first.
  
979373191713
+
At each step, we pick the maximal number that does not yield a contradiction. This immediately gives:
  
Which answer is correct?
+
<cmath>\boxed{619737131179}</cmath>
  
 
== See also ==
 
== See also ==

Latest revision as of 15:30, 3 July 2012

Problem

Find the largest 12-digit number for which every two consecutive digits form a distinct 2-digit prime number.

Solutions

We list all 2 digit primes:

11, 13, 17, 19

23, 29

31, 37

41, 43, 47

53, 59

61, 67

71, 73, 79

83, 89

97


Note that $0,2,4,5,6,8$ can only appear as the first digit. We can construct the solution by appending digits to the right of the current number with the following maps from the current units digit. $1 \to 1,3,7,9; 3 \to 1,7; 7 \to 1,3,9; 9 \to 7$. These alone give a maximum of $11$ digits (counting the first digit of the first prime). Note that $9$ may only map one time, but is mapped to from both $1$ and $7$, so $9$ must also be used as the last digit. To obtain $12$ digits, we must let the first digit must be of the aforementioned alternatives.

We also note that we need to use both the strings $131$ and $737$, since the $3$s may only appear twice. Also, the strings $17$ and $97$ must appear, barring $7$ from being the second digit. These together imply that $3,7,9$ cannot be the first digit, so the second digit must be $1$. We greedily use $61$ first.

At each step, we pick the maximal number that does not yield a contradiction. This immediately gives:

\[\boxed{619737131179}\]

See also

2005 PMWC (Problems)
Preceded by
Problem T9
Followed by
Last Question
I: 1 2 3 4 5 6 7 8 9 10 11 12 13 14 15
T: 1 2 3 4 5 6 7 8 9 10